Fiche de mathématiques
> >

Bac ES-L Obligatoire et Spécialité

Liban 2019

Partager :
4 points

exercice 1 Commun à tous les candidats

Bac ES-L Obligatoire et Spécialité Liban 2019 : image 9


5 points

exercice 2 Candidats de la série ES n'ayant pas suivi l'enseignement de spécialité et candidats de la série L

Bac ES-L Obligatoire et Spécialité Liban 2019 : image 2

Bac ES-L Obligatoire et Spécialité Liban 2019 : image 4


5 points

exercice 2 Candidats de la série ES ayant suivi la spécialité

Bac ES-L Obligatoire et Spécialité Liban 2019 : image 5

Bac ES-L Obligatoire et Spécialité Liban 2019 : image 1

Bac ES-L Obligatoire et Spécialité Liban 2019 : image 10


6 points

exercice 3 Commun à tous les candidats

Bac ES-L Obligatoire et Spécialité Liban 2019 : image 8

Bac ES-L Obligatoire et Spécialité Liban 2019 : image 11

Bac ES-L Obligatoire et Spécialité Liban 2019 : image 7


5 points

exercice 4 Commun à tous les candidats

Bac ES-L Obligatoire et Spécialité Liban 2019 : image 6

Bac ES-L Obligatoire et Spécialité Liban 2019 : image 3





Bac ES-L Obligatoire et Spécialité Liban 2019

Partager :



4 points

exercice 1 : Commun à tous les candidats

1.  Affirmation 1 : y = x - 2 est l'équation réduite de la tangente à \mathscr{C}_u au point d'abscisse 1.  {\blue{\mathbf{Affirmation\ vraie.}}}

Une équation de la tangente à la courbe \mathscr{C}_u au point d'abscisse 1 est de la forme y=u'(1)(x-1)+u(1).

u(x)=3\ln(x)-2x+1\Longrightarrow u'(x)=3\times\dfrac{1}{x}-2 \\\phantom{u(x)=3\ln(x)-2x+1}\Longrightarrow u'(x)=\dfrac{3}{x}-2 \Longrightarrow u'(1)=\dfrac{3}{1}-2=3-2 \\\phantom{u(x)=3\ln(x)-2x+1\Longrightarrow u'(x)=\dfrac{3}{x}-2} \Longrightarrow \boxed{u'(1)=1} \\\\u(x)=3\ln(x)-2x+1\Longrightarrow u(1)=3\ln(1)-2\times1+1 \\\phantom{u(x)=3\ln(x)-2x+1}\Longrightarrow u(1)=3\times0-2+1 \\\phantom{u(x)=3\ln(x)-2x+1}\Longrightarrow\boxed{ u(1)=-1}

D'où une équation de la tangente à la courbe \mathscr{C}_u au point d'abscisse 1 est y=1\times(x-1)-1, soit \boxed{y=x-2}

2.  Affirmation 2 : f est une fonction de densité sur l'intervalle [e ; e²].  {\blue{\mathbf{Affirmation\ vraie.}}}

La fonction f est définie sur l'intervalle [e ; e²] par f(x)=\dfrac{1}{\text{e}^2}\,\ln(x).
f est une fonction de densité sur l'intervalle de réels I = [e ; e²] si f est définie sur I, continue et positive sur I et est telle que \int\limits_{\text{e}}^{\text{e}^2}f(x)\,dx=1.

f est définie sur [e ; e²] par définition.
f est continue sur [e ; e²] comme multiple de la fonction ln qui est continue sur [e ; e²].
f est positive sur [e ; e²] car \dfrac{1}{\text{e}^2}>0 et la fonction ln est positive sur l'intervalle [1 ; +infini[.
Montrons que \int\limits_{\text{e}}^{\text{e}^2}f(x)\,dx=1.
Nous savons par l'énoncé que la fonction F définie sur [e ; e²] par F(x)=x\ln(x)-x est une primitive de la fonction  x\mapsto \ln(x)  sur [e ; e²].

\text{D'où }\ \int\limits_{\text{e}}^{\text{e}^2}f(x)\,dx= \int\limits_{\text{e}}^{\text{e}^2}\dfrac{1}{\text{e}^2}\,\ln(x)\,dx= \dfrac{1}{\text{e}^2}\int\limits_{\text{e}}^{\text{e}^2}\,\ln(x)\,dx=\dfrac{1}{\text{e}^2}\left[\overset{}{F(x)}\right]\limits_{\text{e}}^{\text{e}^2} \\\\\phantom{..........................}=\dfrac{1}{\text{e}^2}\left[\overset{}{F(\text{e}^2)-F(\text{e})}\right] \\\\\phantom{..........................}=\dfrac{1}{\text{e}^2}\left[\overset{}{[\text{e}^2\ln(\text{e}^2)-\text{e}^2]-[\text{e}\ln(\text{e})-\text{e}]}\right] \\\\\phantom{..........................}=\dfrac{1}{\text{e}^2}\left[\overset{}{(\text{e}^2\times2-\text{e}^2)-(\text{e}\times1-\text{e})}\right]=\dfrac{1}{\text{e}^2}\left[\overset{}{(2\text{e}^2-\text{e}^2)-(\text{e}-\text{e})}\right] \\\\\phantom{..........................}=\dfrac{1}{\text{e}^2}\times\text{e}^2=1 \\\\\Longrightarrow\boxed{\ \int\limits_{\text{e}}^{\text{e}^2}f(x)\,dx=1}
Puisque les quatre conditions sont réalisées, nous en déduisons que f est une fonction de densité sur l'intervalle [e ; e²].

3.  Affirmation 3 : La fonction G  définie sur R par  G(x)=-6\,\text{e}^{-2x+1}+6  est la primitive de la fonction g  qui s'annule en \dfrac{1}{2}{\blue{\mathbf{Affirmation\ fausse.}}}

La fonction définie par x\mapsto \text{e}^{-2x+1} est dérivable sur R comme composée de deux fonctions dérivables sur R.
Nous en déduisons que la fonction G  est dérivable sur R.

G'(x)=-6\times(\text{e}^{-2x+1})'+6' \\\phantom{G'(x)}=-6\times(-2x+1)'\text{e}^{-2x+1}+0 \\\phantom{G'(x)}=-6\times(-2)\text{e}^{-2x+1} \\\phantom{G'(x)}=12\text{e}^{-2x+1} \\\\\Longrightarrow\boxed{G'(x)\ {\red{\neq}}\ g(x)}

Par conséquent, la fonction G  n'est pas une primitive de la fonction g.

4.  Affirmation 4 : La fonction h est concave sur l'intervalle [-8 ; -0,75].  {\blue{\mathbf{Affirmation\ vraie.}}}

Etudions le signe de la dérivée seconde h" sur l'intervalle [-8 ; -0,5].

h(x)=\dfrac{4x+1}{x^2}\Longrightarrow h'(x)=\dfrac{(4x+1)'\times x^2-(4x+1)\times (x^2)'}{(x^2)^2} \\\\\phantom{h(x)=\dfrac{4x+1}{x^2}}\Longrightarrow h'(x)=\dfrac{4\times x^2-(4x+1)\times (2x)}{x^4} \\\\\phantom{h(x)=\dfrac{4x+1}{x^2}}\Longrightarrow h'(x)=\dfrac{4x^2-8x^2-2x}{x^4}=\dfrac{-4x^2-2x}{x^4} \\\\\phantom{h(x)=\dfrac{4x+1}{x^2}}\Longrightarrow h'(x)=\dfrac{x(-4x-2)}{x^4} \\\\\phantom{.......................}\Longrightarrow h'(x)=\dfrac{-4x-2}{x^3}

h'(x)=\dfrac{-4x-2}{x^3}\Longrightarrow h''(x)=\dfrac{(-4x-2)'\times x^3-(-4x-2)\times(x^3)'}{(x^3)^2} \\\\\phantom{h'(x)=\dfrac{-4x-2}{x^3}}\Longrightarrow h''(x)=\dfrac{(-4)\times x^3-(-4x-2)\times 3x^2}{x^6} \\\\\phantom{h'(x)=\dfrac{-4x-2}{x^3}}\Longrightarrow h''(x)=\dfrac{-4x^3+12x^3+6x^2}{x^6}=\dfrac{8x^3+6x^2}{x^6} \\\\\phantom{h'(x)=\dfrac{-4x-2}{x^3}}\Longrightarrow h''(x)=\dfrac{2x^2(4x+3)}{x^6} \\\\\phantom{h'(x)=\dfrac{-4x-2}{x^3}}\Longrightarrow\boxed{h''(x)=\dfrac{2(4x+3)}{x^4}}

Puisque 2 > 0 et x4 > 0 sur l'intervalle [-8 ; -0,5], le signe de la dérivée seconde h" est le signe de 4x + 3.

\begin{matrix}4x+3=0\Longleftrightarrow4x=-3\\\phantom{...............}\Longleftrightarrow x=-\dfrac{3}{4}\\\\\phantom{4x+3=0....}\Longleftrightarrow x=-0,75\end{matrix}\ \ \begin{matrix}|\\|\\|\\|\\| \end{matrix}\begin{matrix}4x+3<0\Longleftrightarrow4x<-3\\\phantom{4x+3<0}\Longleftrightarrow x<-\dfrac{3}{4}\\\\\phantom{4x+3<0....}\Longleftrightarrow x<-0,75\end{matrix}\ \ \begin{matrix}|\\|\\|\\|\\| \end{matrix}\begin{matrix}4x+3>0\Longleftrightarrow4x>-3\\\phantom{4x+3>0}\Longleftrightarrow x>-\dfrac{3}{4}\\\\\phantom{4x+3>0....}\Longleftrightarrow x>-0,75\end{matrix}

\text{Donc }x\in[-8\,;-0,75]\Longrightarrow x<-0,75 \\\phantom{\text{Donc }x\in[-8\,;-0,5]}\Longrightarrow 4x+3<0 \\\phantom{\text{Donc }x\in[-8\,;-0,5]}\Longrightarrow h''(x)<0 \\\phantom{\text{Donc }x\in[-8\,;-0,5]}\Longrightarrow \boxed{\text{la fonction }h\ \text{est concave sur [-8 ; -0,75]}}

5 points

exercice 2 : Candidats de la série ES n'ayant pas suivi l'enseignement de
                               spécialité et candidats de la série L

Tableau donnant les estimations du nombre de chenilles de Pyrale dans un camping d'Ardèche :

          \begin{array}{|c|c|c|c|}\hline \text{Date}&01/06/18&02/06/18&03/06/18 \\\hline n&0&1&2\\\hline \text{Nombre de chenilles en centaines}&97&181&258\\\hline \end{array}

Partie 1 : Modèle 1

Dans cette partie, on modélise le nombre de chenilles le n-ième jour après le 1er juin 2018 (nombre exprimé en centaines) par une suite géométrique (un ) de raison q = 1,63. Ainsi u0 = 97.

{\red{1.\ }}u_1=1,63\times u_0 \\\phantom{{\red{1.\ }}u_1}=1,63\times 97 \\\phantom{{\red{1.\ }}u_1}=158,11 \\\\\phantom{{\red{1.\ }}}u_2=1,63\times u_1 \\\phantom{{\red{1.\ }}u_2}=1,63\times 158,11 \\\phantom{{\red{1.\ }}u_2}=257,7193 \\\\\Longrightarrow\boxed{u_2\approx258\ \ \ (\text{arrondi à l'unité})}

2.  Le terme général de la suite (un ) est  u_n=u_0\times q^n , soit  \boxed{u_n=97\times1,63^n}

3.  Montrons que la suite (un ) est croissante.

u_{n+1}-u_n=97\times1,63^{n+1}-97\times1,63^{n} \\\phantom{u_{n+1}-u_n}={\blue{97\times1,63^{n}}}\times1,63-{\blue{97\times1,63^{n}}} \\\phantom{u_{n+1}-u_n}={\blue{97\times1,63^{n}}}(1,63-1) \\\phantom{u_{n+1}-u_n}=97\times1,63^{n}\times0,63 \\\phantom{u_{n+1}-u_n}=61,11\times1,63^{n} \\\\\Longrightarrow u_{n+1}-u_n>0
D'où la suite (un ) est croissante.

Remarque : Nous aurions également pu déduire la croissance de la suite (un ) en remarquant que le premier terme u0 = 97 est strictement positif et que la raison q = 1,63 est strictement supérieure à 1.

4.   Le rang correspondant au 13 juin 2018 est 12.
\overset{.}{u_{12}=97\times1,63^{12}\approx34\ 121.}
Par conséquent, selon ce modèle, nous pouvons estimer que le 13 juin 2018, il y a 3 412 100 chenilles (arrondi à la centaine).

Partie 2 : Modèle 2

Dans cette partie, on modélise le nombre de chenilles le n-ième jour après le 1er juin 2018 (nombre exprimé en centaines) par une suite (vn ) telle que : v0 = 97 et, pour tout entier naturel n, vn+1 = 0,91vn + 93.

1.   L'énoncé nous indique que pour tout entier naturel n : v_n=\dfrac{1}{3}(-2809\times0,91^n+3100).
Le rang correspondant au 13 juin 2018 est 12.
\overset{.}{v_{12}=\dfrac{1}{3}(-2809\times0,91^{12}+3100)\approx731.}
Par conséquent, selon ce modèle, nous pouvons estimer que le 13 juin 2018, il y a 73 100 chenilles (arrondi à la centaine).

2.  Montrons que la suite (vn ) est croissante.

v_{n+1}-v_n=\dfrac{1}{3}(-2809\times0,91^{n+1}+3100)-\dfrac{1}{3}(-2809\times0,91^n+3100) \\\\\phantom{v_{n+1}-v_n}=\dfrac{1}{3}(-2809\times0,91^{n+1}+3100+2809\times0,91^n-3100) \\\\\phantom{v_{n+1}-v_n}=\dfrac{1}{3}(-2809\times0,91^{n+1}+2809\times0,91^n) \\\\\phantom{v_{n+1}-v_n}=\dfrac{2809}{3}(-0,91^{n+1}+0,91^n) =\dfrac{2809}{3}(-0,91^{n}\times0,91+0,91^n) \\\\\phantom{v_{n+1}-v_n}=\dfrac{2809}{3}\times0,91^n(-0,91+1) =\dfrac{2809}{3}\times0,91^n\times0,09 \\\\\phantom{v_{n+1}-v_n}=84,27\times0,91^n \\\\\Longrightarrow\boxed{v_{n+1}-v_n>0}
Par conséquent, la suite (vn ) est croissante.

Partie 3 : Comparaison des différents modèles

La valeur relevée dans le camping le 13 juin 2018 est de 745 centaines de chenilles.

1.  745 centaines de chenilles = 74 500 de chenilles.

Nous avons montré que nous pouvons estimer que le 13 juin 2018, il y a 3 412 100 chenilles selon le modèle 1 et 73 100 chenilles selon le modèle 2.
D'où le modèle 2 paraît être le modèle le plus adapté.

2. a)   Résolvons l'inéquation : vn supegal 1000.

v_n\ge1000\Longleftrightarrow \dfrac{1}{3}(-2809\times0,91^n+3100)\ge1000 \\\\\phantom{v_n\ge1000}\Longleftrightarrow -2809\times0,91^n+3100\ge3000 \\\\\phantom{v_n\ge1000}\Longleftrightarrow -2809\times0,91^n\ge-100 \\\\\phantom{v_n\ge1000}\Longleftrightarrow0,91^n\le\dfrac{-100}{ -2809}\Longleftrightarrow0,91^n\le\dfrac{100}{2809} \\\\\phantom{v_n\ge1000}\Longleftrightarrow\ln(0,91^n)\le\ln(\dfrac{100}{ 2809}) \\\\\phantom{v_n\ge1000}\Longleftrightarrow n\times\ln(0,91)\le\ln(\dfrac{100}{ 2809}) \\\\\phantom{v_n\ge1000}\Longleftrightarrow n\ge\dfrac{\ln(\dfrac{100}{ 2809})}{\ln(0,91)}\ \ \ \ \text{(Changement du sens de l'inégalité car }\ln(0,91)<0)  \\\\\text{Or }\ \dfrac{\ln(\dfrac{100}{ 2809})}{\ln(0,91)}\approx35,366

Puisque n  est un nombre entier naturel, l'inéquation est vérifiée pour n supegal 36.
Par conséquent, l'ensemble des solutions de l'inéquation est l'ensemble des nombres naturels appartenant à l'intervalle [36 ; +infini[.

2. b)   Dans le contexte de l'énoncé, nous pouvons donc prévoir que selon le deuxième modèle, le nombre de chenilles sera au moins égal à 100 000 à partir du 7 juillet 2018 (soit 36 jours après le 1er juin 2018).

5 points

exercice 2 : Candidats de la série ES ayant suivi la spécialité

Partie 1

1.  Graphe probabiliste représentant la situation :
Parmi les clients ayant choisi le plat A :
30 % reprennent le plat A le lendemain,
50 % prennent le plat B le lendemain.
Donc
20 % prennent le plat C le lendemain.

Parmi les clients ayant choisi le plat B :
30 % reprennent le plat B le lendemain,
60 % prennent le plat A le lendemain.
Donc
10 % prennent le plat C le lendemain.

Parmi les clients ayant choisi le plat C :
35 % prennent le plat A le lendemain
45 % prennent le plat B le lendemain,
Donc
20 % reprennent le plat C le lendemain.
Bac ES-L Obligatoire et Spécialité Liban 2019 : image 13


2.  La matrice de transition M  du graphe probabiliste dans l'ordre A-B-C est  \boxed{M=\begin{pmatrix}0,3&0,5 & 0,2\\0,6&0,3&0,1\\0,35&0,45&0,2\end{pmatrix}}

3.  Le restaurateur a noté que le premier jour 35,5 % des clients ont pris le plat A, 40,5 % ont pris le plat B et 24% ont pris le plat C.
Si pour tout entier naturel n  non nul, nous notons  P_n=\begin{pmatrix}a_n & b_n & c_n\end{pmatrix}  la matrice exprimant l'état du nième jour, alors  P_1=\begin{pmatrix}a_1 & b_1 & c_1\end{pmatrix}=\begin{pmatrix}0,355 & 0,405&0,24\end{pmatrix}.

\text{D'où  }P_2=P_1\times M\\\phantom{\text{D'où  }P_2}=\begin{pmatrix}0,355 & 0,405&0,24\end{pmatrix}\times\begin{pmatrix}0,3&0,5 & 0,2\\0,6&0,3&0,1\\0,35&0,45&0,2\end{pmatrix}\\\\\phantom{\text{D'où  }P_2}=\begin{pmatrix}0,355\times0,3+0,405\times0,6+...&0,355\times0,5+0,405\times0,3+...&0,355\times0,2+...\end{pmatrix}\\\\\phantom{\text{D'où  }P_2}=\begin{pmatrix}0,4335 & 0,407&0,1595\end{pmatrix}\\\\\Longrightarrow\boxed{P_2=\begin{pmatrix}0,4335 & 0,407&0,1595\end{pmatrix}}

4.   Calculons P12 et P13.

P_{12}=P_1\times M^{11}\\\phantom{P_{12}}=\begin{pmatrix}0,355 & 0,405&0,24\end{pmatrix}\times\begin{pmatrix}0,3&0,5 & 0,2\\0,6&0,3&0,1\\0,35&0,45&0,2\end{pmatrix}^{11}\\\phantom{P_{12}}\approx\begin{pmatrix}0,355 & 0,405&0,24\end{pmatrix}\times\begin{pmatrix}0,431&0,410 & 0,159\\0,431&0,410 & 0,159\\0,431&0,410 & 0,159\end{pmatrix}\\\\\Longrightarrow\boxed{P_{12}\approx\begin{pmatrix}0,431 & 0,410&{\red{0,159}}\end{pmatrix}}

D'où la proportion de clients ayant choisi le plat C le 12ième jour est  c_{12}\approx0,159 , soit environ 15,9 %.

P_{13}=P_1\times M^{12}\\\phantom{P_{13}}=\begin{pmatrix}0,355 & 0,405&0,24\end{pmatrix}\times\begin{pmatrix}0,3&0,5 & 0,2\\0,6&0,3&0,1\\0,35&0,45&0,2\end{pmatrix}^{12}\\\phantom{P_{13}}\approx\begin{pmatrix}0,355 & 0,405&0,24\end{pmatrix}\times\begin{pmatrix}0,431&0,410 & 0,159\\0,431&0,410 & 0,159\\0,431&0,410 & 0,159\end{pmatrix}\\\\\Longrightarrow\boxed{P_{13}\approx\begin{pmatrix}0,431 & 0,410&{\red{0,159}}\end{pmatrix}}
D'où la proportion de clients ayant choisi le plat C le 13ième jour est  c_{13}\approx0,159 , soit environ 15,9 %.

Par conséquent, le restaurateur a raison en affirmant que le 12ième jour, la proportion de clients qui choisiront le plat C sera à peu près la même que le 13ième jour, soit environ 15,9 %.

Partie 2

1. a)  Montrons que le graphe est connexe et qu'il admet une chaîne eulérienne.

Puisqu'il existe une chaîne entre n'importe quelle paire de sommets distincts du graphe, ce graphe est connexe.
Or un graphe connexe contient une chaîne eulérienne si et seulement si il possède 0 ou 2 sommets de degré impair.
Voici un tableau indiquant les degrés des sommets du graphe :

              \begin{array}{|c|c|c|c|c|c|c|c|c|}\hline \text{Sommets}&H_1&H_2&H_3&H_4&H_5&H_6&H_7&H_8\\\hline \text{Degrés}&{\red{3}}&4&6&2&2&{\red{3}}&4&2\\\hline \end{array}

Comme il n'y a que deux sommets de degré impair (les sommets H1 et H6), ce graphe connexe contient une chaîne eulérienne.
Par conséquent, il existe un parcours empruntant toutes les rues une et une seule fois.

1. b)  Un parcours empruntant toutes les rues une et une seule fois peut partir de H1 et y revenir si le graphe contient un cycle eulérien.
Or un graphe connexe contient un cycle eulérien si et seulement si il ne possède aucun sommet de degré impair.
Comme il y a deux sommets de degré impair (les sommets H1 et H6), ce graphe connexe ne contient pas de cycle eulérien.
Par conséquent, il n'existe aucun parcours empruntant toutes les rues une et une seule fois partant de H1 et y revenant.

2.   Utilisons l'algorithme de Dijkstra afin de déterminer le temps minimal pour aller de H4 à H8.

           \begin{array}{|c|c|c|c|c|c|c|c|c|} \hline H_1&H_2&H_3&H_5&H_6&H_7&H_8& \text{Sommet sélectionné}  \\\hline8_{H_4}&\infty&\infty&15_{H_4}&\infty&\infty&\infty&H_4\\\hline\cellcolor{magenta}&17_{H_1}&24_{H_1}&15_{H_4}&\infty&\infty&\infty&H_1\\\hline\cellcolor{magenta}&17_{H_1}&22_{H_5}&\cellcolor{magenta}&\infty&\infty&\infty&H_5\\\hline\cellcolor{magenta} & \cellcolor{magenta}&22_{H_5}&\cellcolor{magenta}&34_{H_2}&28_{H_2}&\infty&H_2\\\hline\cellcolor{magenta}&\cellcolor{magenta}&\cellcolor{magenta} &\cellcolor{magenta}  &27_{H_3}&26_{H_3}&50_{H_3}&H_3\\\hline\cellcolor{magenta}&\cellcolor{magenta}&\cellcolor{magenta}&\cellcolor{magenta}&27_{H_3}&\cellcolor{magenta}&35_{H_7}&H_7\\\hline\cellcolor{magenta}&\cellcolor{magenta}&\cellcolor{magenta}&\cellcolor{magenta}&\cellcolor{magenta}&\cellcolor{magenta}&35_{H_7}&H_6\\\hline \end{array}

D'où le trajet le plus rapide pour aller de H4 à H8 est H4 - H5 - H3 - H7 - H8.
Ce trajet durera 35 minutes.

6 points

exercice 3 : Commun à tous les candidats

Soit  f(x)=1+(-4x^2-10x+8)\,\text{e}^{-0,5x}\ \ \ \ \ (x\in[-4\,;10])

1.  La fonction définie par x\mapsto (-4x^2-10x+8)\,\text{e}^{-0,5x} est dérivable sur [-4 ; 10] comme produit de deux fonctions dérivables sur [-4 ; 10].
Donc la fonction f  est dérivable sur [-4 ; 10] comme somme de deux fonctions dérivables sur [-4 ; 10].

f'(x)=1'+(-4x^2-10x+8)'\times\,\text{e}^{-0,5x}+(-4x^2-10x+8)\times\,(\text{e}^{-0,5x})' \\\\\phantom{f'(x)}=0+(-8x-10)\times\text{e}^{-0,5x}+(-4x^2-10x+8)\times(-0,5x)'\,\text{e}^{-0,5x}  \\\\\phantom{f'(x)}=(-8x-10)\times\text{e}^{-0,5x}+(-4x^2-10x+8)\times(-0,5)\,\text{e}^{-0,5x} \\\\\phantom{f'(x)}=(-8x-10+2x^2+5x-4)\,\text{e}^{-0,2x} \\\\\Longrightarrow\boxed{f'(x)=(2x^2-3x-14)\,\text{e}^{-0,5x}}

2.  Puisque l'exponentielle est strictement positive sur R et a fortiori sur [-4 ; 10], le signe de la dérivée f' (x ) sera le signe du polynôme du second degré 2x^2-3x-14.

Le discriminant deltamaj = (-3)2 - 4 multiplie 2 multiplie (-14) = 9 + 112 = 121 > 0.
Puisque le discriminant du trinôme est strictement positif, ce trinôme admet deux racines distinctes :

x_1=\dfrac{3-\sqrt{121}}{2\times2}=\dfrac{3-11}{4}=\dfrac{-8}{4}=-2\Longrightarrow\boxed{x_1=-2\in[-4\,;10]} \\\\x_2=\dfrac{3+\sqrt{121}}{2\times2}=\dfrac{3+11}{4}=\dfrac{14}{4}=3,5\Longrightarrow\boxed{x_2=3,5\in[-4\,;10]}

Le signe du coefficient de x² étant positif, le polynôme du second degré 2x^2-3x+-14 est positif à l'extérieur des racines et négatif entre les racines.

D'où le tableau de signes de  2x^2-3x+-14 sur [-4 ; 10] :

              \begin{array}{|c|ccccccc|}\hline x&-4&&-2&&3,5&&10 \\\hline2x^2-3x-14&&+&0&-&0&+&\\\hline \end{array}

Nous en déduisons le tableau de signes de f' (x ) et les variations de la fonction f .

              \begin{array}{|c|ccccccc|}\hline &&&&&&&\\ x&-4&&-2&&3,5&&10\\&&&&&&&\\\hline &&&&&&&\\ f'(x)&&+&0&-&0&+&\\&&&&&&&\\\hline &&&1+12\,\text{e}\approx33,6&&&&1-492\ \text{e}^{-5} \\ f(x)&&\nearrow&&\searrow&&\nearrow& \\ &1-16\,\text{e}^{2}\approx-117,2&&&&1-76\,\text{e}^{-1,75}&&\\\hline \end{array}

3. a)   La fonction f  est continue et strictement croissante sur [-4 ; -2].
f (-4) environegal -117,2 < 0 et f (-2) environegal 33,6 > 0.
Donc 0 est compris entre f (-4) et f (-2).
D'après le corollaire du théorème des valeurs intermédiaires, l'équation f (x ) = 0 admet une unique solution alpha appartenant à l'intervalle [-4 ; -2].

3. b)  Tableau reprenant les deux premiers passages dans la boucle de l'algorithme proposé.

          \begin{array}{|c|c|c|c|c|c|c|}\hline &m&\text{signe de }p&a&b&b-a&b-a>10^{-1}\\\hline\text{Initialisation}&\cellcolor{gray!40}&\cellcolor{gray!40}&-4&-2&2&\text{VRAI }\\\hline\text{Après le 1}^\text{er} \text{passage}&&&&&&&&-3&\text{Négatif}&-4&-3&1&\text{VRAI }\\\text{dans la boucle}&&&&&&\\\hline\text{Après le 2}^\text{ème} \text{passage}&&&&&&&&\red{-3,5}&\red{\text{Positif}}&\red{-3,5}&\red{-3}&\red{0,5}&\red{\text{VRAI }}\\\text{dans la boucle}&&&&&&\\\hline \end{array}

3. c)  Cet algorithme permet de résoudre l'équation f (x ) = 0 et de trouver un encadrement d'amplitude maximale de 10-1 comprenant l'unique solution alpha appartenant à l'intervalle [-4 ; -2].

Nous avons donc : \boxed{-3,1875\le\alpha\le-3,125}

4.  Selon l'énoncé, nous admettons qu'une primitive de la fonction f  sur l'intervalle [-4 ; 10] est la fonction F  définie par  F(x)=x+(8x^2+52x+88)\,\text{e}^{-0,5x}.

La valeur moyenne de la fonction f  dans l'intervalle [-4 ; 10] est :

\mu=\dfrac{1}{10-(-4)}\,\int\limits_{-4}^{10}f(x)\,dx=\dfrac{1}{14}\,\int\limits_{-4}^{10}f(x)\,dx  \\\\\phantom{\mu}=\dfrac{1}{14}\times\left[\overset{}{F(x)}\right]\limits_{-4}^{10}=\dfrac{1}{14}\times\left[\overset{}{x+(8x^2+52x+88)\,\text{e}^{-0,5x}}\right]\limits_{-4}^{10}  \\\\\phantom{\mu}=\dfrac{1}{14}\times\left[\overset{}{10+(8\times10^2+52\times10+88)\,\text{e}^{-0,5\times10}-(-4)-[8\times(-4)^2+52\times(-4)+88]\,\text{e}^{-0,5\times(-4)}}\right] \\\\\phantom{\mu}=\dfrac{1}{14}\times\left[\overset{}{14+1408\,\text{e}^{-5}-8\,\text{e}^{2}}\right] \\\\\Longrightarrow\boxed{\mu=\dfrac{1}{14}\times\left[\overset{}{14+1408\,\text{e}^{-5}-8\,\text{e}^{2}}\right]\approx-2,54}

5 points

exercice 4 : Commun à tous les candidats

Partie 1

1.   Nous sommes en présence de 300 interrogatoires, ces interrogatoires étant indépendants et identiques.
Lors de chaque interrogatoire, deux issues sont possibles :
le succès : "la personne est respectueuse de son environnement" avec une probabilité p  = 0,72.
l'échec : "la personne n'est pas respectueuse de son environnement" avec une probabilité 1 - p  = 1 - 0,72 = 0,28.
Donc la variable aléatoire X  comptant le nombre de personnes respectueuses de leur environnement suit une loi binomiale de paramètres n  = 300 et p  = 0,72.

2.  Nous devons déterminer   P(X=190).

P(X=190)=\begin{pmatrix}300\\190\end{pmatrix}\times0,72^{190}\times(1-0,72)^{300-190} \\\\\phantom{P(Y=3)}=\begin{pmatrix}300\\190\end{pmatrix}\times0,72^{190}\times0,28^{110}  \\\\\phantom{P(Y=3)}\approx0,0002396367\\\\\Longrightarrow\boxed{P(X=190)\approx0,0002}

Par conséquent, la probabilité que 190 personnes soient respectueuses de leur environnement est environ égale à 0,0002 (arrondie à 10-4).

3.   Nous devons déterminer   P(X\ge220).

P(X\ge220)=1-P(X<220) \\\phantom{P(X\ge220)}=1-P(X\le219) \\\phantom{P(X\ge220)}\approx1-0,67089721 \\\phantom{P(X\ge220)}\approx0,32910279 \\\\\Longrightarrow\boxed{P(X\ge220)=0,3291}

Par conséquent, la probabilité qu'aa moins 220 personnes soient respectueuses de leur environnement est environ égale à 0,3291 (arrondie à 10-4).

Partie 2

1.  Résoudre dans R l'inéquation : 2x2 - 7x - 4 supegal 0.

Etudions le signe du polynôme du second degré 2x2 - 7x - 4.

Le discriminant deltamaj = (-7)2 - 4 multiplie 2 multiplie (-4) = 49 + 32 = 81 > 0.
Puisque le discriminant du trinôme est strictement positif, ce trinôme admet deux racines distinctes :

x_1=\dfrac{7-\sqrt{81}}{2\times2}=\dfrac{7-9}{4}=\dfrac{-2}{4}=-0,5\Longrightarrow\boxed{x_1=-0,5} \\\\x_2=\dfrac{7+\sqrt{81}}{2\times2}=\dfrac{7+9}{4}=\dfrac{16}{4}=4\Longrightarrow\boxed{x_2=4}

Le signe du coefficient de x² étant positif, le polynôme du second degré 2x2 - 7x - 4 est positif à l'extérieur des racines et négatif entre les racines.

D'où le tableau de signes de 2x2 - 7x - 4 sur R :

\begin{array}{|c|ccccccc|}\hline x&-\infty&&-0,5&&4&&+\infty \\\hline 2x^2-7x-4&&+&0&-&0&+&\\\hline \end{array}

Par conséquent, l'ensemble S des solutions de l'inéquation 2x2 - 7x - 4 supegal 0 est \boxed{S=]-\infty\,;-0,5]\cup [4\,;+\infty[}

2.  Nous pouvons modéliser le tirage de ce nombre par la loi uniforme sur l'intervalle [0 ; 10].
Pour être solution de l'inéquation, le nombre tiré doit être compris entre 4 et 10.

Soit Y  la variable aléatoire qui suit la loi uniforme sur l'intervalle [0 ; 10].
Nous devons déterminer P(4\le Y\le10).

P(4\le Y\le10)=\dfrac{10-4}{10-0}=\dfrac{6}{10}=0,6

Par conséquent, la probabilité pour le nombre tiré soit solution de l'inéquation est égale à 0,6.

Partie 3

1.  Soit Z  une variable aléatoire qui suit la loi normale d'espérance mu = 2,3 et d'écart-type sigma = 0,11.

1. a)   Par la calculatrice, nous obtenons :  P(2,18\le Z\le 2,42)\approx0,72.

1. b)   Nous savons que  \overset{.}{P(Z>\mu)=0,5}, soit que  \overset{.}{P(Z>2,3)=0,5.}

Dès lors,  P(Z\ge2,25)=P(2,25\le Z \le2,3)+P(Z> 2,3)\Longleftrightarrow P(Z\ge2,25)=P(2,25\le Z \le2,3)+0,5.

Or, par la calculatrice, nous obtenons :  P(2,25\le Z \le2,3)\approx0,17528185

\text{D'où }\ P(Z\ge2,25)=P(2,25\le Z \le2,3)+0,5\Longrightarrow P(Z\ge2,25)=0,17528185+0,5\\\phantom{\text{D'où }\ P(Z\ge2,25)=P(2,25\le Z \le2,3)+0,5}\Longrightarrow P(Z\ge2,25)=0,67528185\\\phantom{\text{D'où }\ P(Z\ge2,25)=P(2,25\le Z \le2,3)+0,5}\Longrightarrow\boxed{P(Z\ge2,25)\approx0,68}

2.   Nous savons que si Z  suit la loi normale d'espérance mu et d'écart-type sigma,
alors  \overset{.}{P(\mu-2\sigma \le Z\le\mu+2\sigma )\approx0,95.}

\text{Dès lors, }\ P(2,18\le Z \le2,42)\approx0,95\Longleftrightarrow P(2,3-0,12\le Z \le2,3+0,12)\approx0,95 \\\phantom{\text{Dès lors, }\ P(2,18\le Z \le2,42)\approx0,95}\Longleftrightarrow P(\mu-2\times0,06\le Z \le\mu+2\times0,06)\approx0,95 \\\phantom{....................................................................}{\red{[P(\mu-2\times\sigma \ \ \ \le Z\le\ \mu+2\times\sigma )\ \ \ \approx0,95]}}

Nous en déduisons alors que  \boxed{\sigma\approx0,06}

Publié le
ceci n'est qu'un extrait
Pour visualiser la totalité des cours vous devez vous inscrire / connecter (GRATUIT)
Inscription Gratuite se connecter
Merci à
Hiphigenie
/
malou Webmaster
pour avoir contribué à l'élaboration de cette fiche


Vous devez être membre accéder à ce service...

Pas encore inscrit ?

1 compte par personne, multi-compte interdit !

Ou identifiez-vous :


Rester sur la page

Inscription gratuite

Fiches en rapport

parmi 1674 fiches de maths

Désolé, votre version d'Internet Explorer est plus que périmée ! Merci de le mettre à jour ou de télécharger Firefox ou Google Chrome pour utiliser le site. Votre ordinateur vous remerciera !